Wann können wir davon ausgehen, dass die Wellenfunktion separabel ist

Beim Ausarbeiten der stationären Zustände eines einzelnen Teilchens in einer 3D-Box mit unendlichem Potenzial ( v = 0 in einem Quader mit bekannten Abmessungen, v = überall sonst), wurde mir klar, dass ich annehmen musste, dass die Wellenfunktion in ein Produkt aus drei Funktionen zerlegbar ist, ψ ( x , j , z ) = X ( x ) Y ( j ) Z ( z ) , um zu finden ψ . Warum ist das so und unter welchen Bedingungen? Was garantiert mir, dass ich das kann? Der Text, dem ich folge, ist diesbezüglich nicht besonders klar.

Es gibt tatsächlich gute strenge Beweise dafür, dass die Annahme der Trennbarkeit für bestimmte Systeme keine Informationen über das Problem "zerstört", und ich erinnere mich vage, dass sie die Untersuchung der Invarianz des Hamilton-Operators unter der Wirkung von Lie-Gruppen beinhalten, die mit der Symmetrie verbunden sind Gruppe des Systems, aber ich kenne die Einzelheiten ehrlich gesagt nicht. Selbstverständlich vermeiden die meisten Einführungsbücher die Erwähnung solcher Schwierigkeiten vollständig (da der Text sonst einen komplizierten Weg nehmen würde, der pädagogisch selbstmörderisch wäre).
Aha. Ich habe mich gefragt, ob wir dies für fast alle praktischen Anwendungen einfach annehmen können oder ob es pathologische Gegenbeispiele oder überhaupt keine Gegenbeispiele gibt.
Es gibt viele Gegenbeispiele. Sie könnten beispielsweise versuchen anzunehmen, dass das Teilchen in der 2D-Box Eigenzustände hat, die in Polarkoordinaten trennbar sind ( r , ϕ ) , aber das schlägt fehl, weil sich herausstellt, dass die Eigenzustände in diesen Koordinaten nicht trennbar sind. Die meisten prototypischen exakt lösbaren Quantensysteme werden jedoch in einem Koordinatensystem modelliert, in dem das System am Ende trennbar ist, sodass Sie sich keine Sorgen machen müssen, wenn Sie etwas darüber erfahren. Aber im Allgemeinen ist es nicht unbedingt sicher anzunehmen, dass das System in jedem beliebigen Koordinatensystem trennbar ist.
Ein komplizierteres Beispiel für die allgemeine Unfähigkeit, Variablen zu trennen, ist jede Situation, in der die Born-Oppenheimer-Näherung fehlschlägt. Beispielsweise wird in der Molekülphysik oft angenommen, dass es eine Trennbarkeit zwischen den Rotations-, Translations-, Schwingungs-, Elektronen- und Kernspinkoordinaten gibt, aber in Wirklichkeit ist dies eine Näherung, die beispielsweise bei experimentell- beobachteten spektrale Signaturen der rovibronischen Kopplung.
Es ist gerechtfertigt, weil es funktioniert. Beim Lösen einer Differentialgleichung wird die Antwort eindeutig durch die Gleichung und die Randbedingungen bestimmt. Wenn Sie eine Lösung ausprobieren, wenn sie funktioniert, dann akzeptieren Sie sie. @DumpsterDoofus Ihre 2D-Box ist quadratisch oder eine 2D-Kugel?
@luming: Ich habe ein Beispiel für ein 2D-Quadrat gegeben. Sie können die Schrödinger-Gleichung für ein unendliches 2D-Quadrat gut in Polarkoordinaten formulieren, eine Trennung der Variablen annehmen, und es wird fehlschlagen (weil die Wellenfunktionen in Polarkoordinaten nicht trennbar sind). Das OP stimmt zu, dass im Fall der unendlichen quadratischen 3D-Box die Annahme einer Trennung der Variablen zu Lösungen führt, die die Schrödinger-Gleichung erfüllen. Ich nehme an, er macht sich Sorgen, ob es irgendwelche Eigenzustände gibt, die durch die Anwendung dieser Annahme "übersehen" werden.
Allerdings können Sie im Spezialfall des unendlichen 3D-Würfels rigoros beweisen, dass die Trennung der Variablen alle Lösungen ergeben muss, da der Hamilton-Operator die direkte Summe von drei Operatoren ist, was (glaube ich?) bedeutet, dass die Wellenfunktionen tensor- Produkte der Eigenzustände der einzelnen Operatoren, was bedeutet, dass die Eigenzustände kartesisch separierbar sind. Bei komplizierteren Systemen habe ich keine Ahnung, wie das funktioniert (es ist zu kompliziert für mich, um es zu verstehen).
Rod Vance oder einige der anderen Benutzer mit wirklich hohen Wiederholungszahlen könnten es jedoch wissen.
@DumpsterDoofus Im Fall einer unendlichen quadratischen Box dachte ich, wir könnten die Variable in trennen r und θ , erfüllen dann die Randbedingung des quadratischen Kästchens. Wir sollten genau die gleichen Eigenwerte erhalten, wenn wir die kartesischen Koordinaten eingeben. Ist es nicht wahr?
@luming: Nein, das geht gar nicht. Zum Beispiel der Grundzustand n x = 1 , n j = 1 des 2D-Infinity-Quadrat-Well-Kastens mit L x = L j = 1 ist 2 Sünde ( π x ) Sünde ( π j ) , die nicht in das Formular geschrieben werden können EIN ( r ) B ( θ ) . Sie können die Unmöglichkeit durch Einfügen beweisen x r cos ( θ ) , j r Sünde ( θ ) und dann das Ergebnis für fest darstellen r und variabel θ . Es wird niemals für jede Wahl konstant sein r > 0 . Daher schlägt die Trennung der Variablen in diesem Fall vollständig fehl, egal wie sehr Sie es versuchen.
@luming: Außerdem ist der Grundzustand einfach entartet, sodass es keine Möglichkeit gibt, dass innerhalb des entarteten Unterraums eine lineare Kombination existiert, die eine Zerlegung der Form zulässt EIN ( r ) B ( θ ) . Andernfalls gilt für die mehrfach entarteten angeregten Zustände ( n x > 1 oder n j > 1 ), wissen Sie, dass die Wellenfunktion an den Kanten immer die Null-Randbedingung erfüllen muss, aber kombinieren Sie dies mit einer Annahme, dass ψ = EIN ( r ) B ( θ ) bedeutet, dass die Wellenfunktion für alle Null sein muss r > 0,5 , die viele der Punkte in der Box enthält! Dies ist ein Widerspruch.
@luming: Vergiss eigentlich meine Argumentation in dem Satz "Ergebnis fest darstellen r und variabel θ ...", das war falsch. Die richtige Argumentation ist, dass in Polarkoordinaten der Grundzustand gegeben ist durch ψ ( r , θ ) = Sünde ( π r Sünde ( θ ) ) Sünde ( π r cos ( θ ) ) . Allerdings das Verhältnis ψ ( r 1 , θ ) / ψ ( r 2 , θ ) ist nicht unabhängig von θ wann immer r 1 r 2 . Wenn es jedoch trennbar wäre, würden Sie erhalten EIN ( r 1 ) / EIN ( r 2 ) , die konstant ist. Dies ist ein Widerspruch.
@DumpsterDoofus Wie Sie bereits sagten, ist der Hamiltonian die direkte Summe zweier Operatoren p x 2 , p j 2 . Beim Schreiben in Polarkoordinaten kann der Hamiltonoperator jedoch nicht als direkte Summe zweier Operatoren schreiben, in denen jeder von ihnen nur enthalten ist r oder θ . Ich denke, das erklärt, warum wir die Wellenfunktion nicht in die Form wie zerlegen können EIN ( r ) B ( θ ) . Erinnern Sie sich, wie wir das Problem des Wasserstoffatoms lösen, wir können die Wellenfunktion nur so trennen: R ( r ) Y ( θ , ϕ ) , weil wir nur hamiltonisch schreiben können Ö ( r ) + Ö ' ( θ , ϕ )

Antworten (2)

Die Trennung von Variablen ist in der Tat ein heikles Thema in partiellen Differentialgleichungen. Bis heute haben wir (soweit ich weiß) keine vollständige Theorie über die Bedingungen, die dies ermöglichen. Die übliche Haltung besteht darin, Existenz- und Eindeutigkeitstheoreme für die Lösungen einer gegebenen PDE zu haben und einen Ansatz aus der Trennung von Variablen zu verwenden, indem wir eine allgemeine Lösung finden, sollten wir die Lösung wie Luming kommentiert haben.

Soweit ich weiß, haben wir für bestimmte Fälle eine strenge Rechtfertigung für die Verwendung der Trennung von Variablen in gegebenen Koordinaten, die sich auf die auf die PDE wirkende Symmetriegruppe beziehen (wie BumbsterDoofus auch in Kommentaren sagte). Ein (etwas altes) Buch, das dies erklärt, ist Millers "Symmetry and Separation of Variables", das Sie hier online finden können http://www.ima.umn.edu/~miller/separationofvariables.html . Wie es im Vorwort heißt, wissen wir zwar, wie wir einige PDEs (insbesondere niederdimensionale) begründen können, aber wir haben keine vollständige Theorie für alle Differentialgleichungen, die wir betrachten möchten (z. B. die dreidimensionale Wellengleichung). Weitere Entwicklungen über Millers Buch hinaus sind mir nicht bekannt, aber ich habe danach gesucht und keine entscheidenden Änderungen gefunden (das könnte aber an meiner Unkenntnis liegen).

Solange Sie gebundene Zustände in Betracht ziehen, sollten Sie sich meiner Meinung nach auf jeden Fall keine Sorgen um diese Dinge machen. Die Existenz- und Eindeutigkeitstheoreme in Verbindung mit der Möglichkeit, eine allgemeine Lösung bereitzustellen, sollten ausreichen (ich bin immer misstrauisch gegenüber Streuung Zustände, weil sie nicht quadratintegrierbar sind und subtiler sein könnten). Wenn Sie mit dieser Antwort nicht zufrieden sind, wäre es wahrscheinlich eine großartige Frage zu Math Stackexchange, nach dem Status der Trennung von Variablen zu fragen, obwohl ich denke, dass sich die Antwort sowieso auf die Symmetriegruppe der fraglichen PDE bezieht und könnte für Ihren Kontext übertrieben sein.

Die Logik geht wie folgt.

Wir können Lösungen in Form von erraten X ( x ) Y ( j ) Z ( z ) für ein Teilchen in einer dreidimensionalen Box. Wir können solche Lösungen finden. Die Frage ist, übersehen wir irgendeine Lösung?

Die Funktion X ( x ) Eigenfunktion des selbstadjungierten Operators ist

(1) H x = 1 2 2 x 2 + v ( x )
v ( x ) ist das Potenzial der Unendlichkeitswand. Und das gleiche für j , z . Damit bilden sie unter geeigneten Randbedingungen einen vollständigen Satz von Funktionen. Wir erweitern die allgemeine Form der Lösung als
(2) ψ ( x , j , z ) = l m n c l m n X l ( x ) Y m ( j ) Z n ( z )

Seit [ H x , H ] = 0 , bildet die Eigenfunktion eines Teilchens im dreidimensionalen Kasten einen simultanen Eigenzustand als Eigenfunktion von H x . Wir können fallen l und die l Abhängigkeit im Ausdehnungskoeffizienten c c l m in Gl. (2). Gleiches gilt für j und z . Daher kann die Eigenfunktion des Teilchens im dreidimensionalen Kasten geschrieben werden als X ( x ) Y ( j ) Z ( z ) .